Talk:2018 AMC 12B Problems/Problem 13

Remark (Coordinate Geometry)

Alternatively, use the Shoelace Theorem once you have found the coordinates in terms of $x$ and $y$. The $x$s and $y$s will conveniently cancel and the answer is (still) $\boxed{\textbf{C}}$.

~Fasolinka